nyyun928 - Cardiovascular System

Pataasin ang iyong marka sa homework at exams ngayon gamit ang Quizwiz!

Case Studv Questions 65-68 refer to the following case study: Mr.L. is admitted to the critical care unit with dyspnea and shortness of breath on exertion. He has a history of chronic obstructive pulmonary disease (COPD). Chest X-ray confirms a diagnosis of congestive heart failure. A pulmonary artery catheter is inserted

...

Case Study Mr.S., a 54-year-old man with a known history of coronary artery disease, is admitted with progressively worsening angina pectoris. Tests were done to delineate the extent of his disease and to deter mine if he is a candidate for coronary artery bypass grafting (CABG).

...

Case Study Questions 116-119 refer to the following study: Mr.M a 26-year-old man, sustained severe abdominal injuries in an automobile accident. When he is admitted to the intensive care unit (!CU), his blood pressure is 80/50 mm Hg, his heart rate is 130 heats/minute, and his respiratory rate is 30 breaths/minute.

...

Case Study Questions 124-127 refer to the following case study: Mr.B. presents with chest pain that lasts more than 30 minutes and that is unresponsive to sublingual nitroglycerin. In addition, ST elevation is present in leads II, III and aVF. Thrombolytic therapy is instituted

...

Case Study Questions 128-131 continue the previous case study. Another thrombolytic therapy that may be instituted for Mr. B. involves tissue plasminogen activator (t-P A).

...

Case Study Questions 132-133 refer to the following case study: Mr. F. has been admitted with an acute anterior wall MI. He continues to have intermittent chest pain relieved with morphine sulfate 5mg. I.V. push. His blood pressure is 130/70 and his pulse is 110 beats per minute.

...

Case Study Questions 134-135 refer to the following case study: Mr. Q. is admitted to the critical care unit with an acute inferior-posterior wall MI. He is having .frequent premature ventricular contractions. You are about to administer lidocaine.

...

Case Study Questions 136-138 refer to the following case study: Mr. Q. is on a 2 mg/minute drip of lidocaine. His cardiac monitor alarm goes off. The monitor shows ventricular tachycardia. He has a pulse but is hemodynamically unstable.

...

Case Study Questions 178-181 refer to the following case study: Mr. F., age 43, is admitted to the ICU. He has had chest pain for the last 2 hours and ST elevation in V1-V4. Acute anterior-wall myocardial infarction is diagnosed. The physician plans to administer t-P A (tissue plasminogen activator)

...

Case Study Questions 182-187 refer to the following case study: Mrs. R., age 52, is admitted to the ICU, awaiting a cardiac transplantation. She has a history of idiopathic cardiomyopathy.

...

Case Study Questions 26-32 refer to the following case study: Mr.S., who has a history of congestive heart failure, is admitted to your unit with a medical diagnosis of sepsis and dehydration.Initial assessment reveals intravascular depletion. A pulmonary artery catheter is inserted.

...

Case Study Questions 47-51 continue the previous case study. The patient, Mr.A, suddenly develops complete heart block. A temporary pacemaker is inserted, and Mr. A. is scheduled for a permanent pacemaker.

...

Case Study Questions 52-61 refer to the following case study: Mr. I. is admitted with an inferior wall infarction. His blood pressure is 110/70 and his pulse is 62. He is presently pain-free.

...

Case Study Questions 71-82 refer to the following case study: Mrs. C. is admitted to the critical care unit with crushing chestpain. An ECG reveals a massive infarction. She is hypotensive,diaphoretic, and oliguric. A diagnosis of cardiogenic shock is made.

...

Case Study Questions 88- 93 refer to the following case study: Mr. A., age 5.J, has a history of angina pectoris. He has been having .frequent chest pain, necessitating increased sublingual doses of nitroglycerin. ST depressions are noted on his ECG.

...

Case Study Questions 96-101 refer to the following case study: Mr.K. , age 39, is admitted to the intensive care unit (ICU) after a motor vehicle accident. He suffered a rib fracture when his chest hit the steering wheel. His diagnosis is pericardial tamponade.

...

Case Study: Mr. L., 66 years old, is admitted with an acute inferior wall MI. Questions 167-173 refer to the above situation.

...

Case Study: Mr. V., 57, is admitted to the ICU with a diagnosis of acute lateral wall myocardial infarction. Upon admission to the ICU, the nurse assesses him to be quite anxious. Questions 160 -166 refer to the above case study.

...

CaseStudv Questions 42-46 refer to the following case study: Mr.A. is admitted to the critical care unit with an acute anterior wall myocardial infarction (MI). A Swan-Ganz catheter is inserted

...

86. The intra-aortic balloon pump is: A. Deflated during systole B. Inflated during systole C. Deflated during diastole D. None of the above

Correct answer - A The intra-aortic balloon pump is deflated during systole and createsa potentially empty space to accommodate, without resistance, the stroke volume. This decreases impedance to left ventricular ejection (afterload) and decreases left ventricular work load.

126. Thrombolytic therapy is contraindicated in a recent: A. Deep vein thrombosis B. Pulmonary emboli C. Cerebrovascular accident (CV A) D. Coronary occlusion

Corrcct answer-C Streptokinase is contraindicated in the patient with a recent cardiovascular accident (CVA), active Gl bleeding, coagulation disorder, cardiac trauma or massage, intracranial neoplasm, severe hypertension, mitral valve disease with atrial fibrillation, subacute bacterial endocarditis, or acute or chronic renal or hepatic failure: streptokinase is also contraindicated in the patientwhohas undergone surgery within the past 10 days (including paracentesis, thoracentesis, and invasive biopsy). To avoid complications, a thorough assessment- including a detailed history, a physical examination, and laboratory studies- should be obtained before starting therapy. Anticoagulant or anti platelet agents should not be given concurrently. Prothrombin time, partial thromboplastin time, thrombin time, and total platelet count should be measured before and during streptokinase.

66. Which is not an early sign of CHF? A. Increase in central venous pressure (CVP) reading B. Fine, moist crackles at lung bases C. s3 heart sound D. Tachycardia

Correct answer - A A diseased left ventricle cannot pump blood returning from the lungs into systemic circulation; this decreases cardiac output, and the patient's heart rate increases to compensate. An S3 may appear, indicating heart failure. Lung pressure increases because of blood accumulation, and crackles may be auscultated. Increased central venous pressure (CVP) is a late sign of left ventricular failure. As lung pressure continues to increase, producing pulmonary edema, Increases in right ventricular pressure produce an elevated CVP.

67. The clinical presentation of Mr.L. with left ventricular failure includes: A. Nocturnal dyspnea.... B. Bounding pulses....... C. Hepatosplenomegaly D. Pitting edema...........

Correct answer - A Clinical presentation of a patient with left ventricular failure would include nocturnal dyspnea, along with other signs and symptoms, such as tachycardia, diaphoresis, crackles, pallor, gallop rhythm, and elevated pulmonary artery diastolic and pulmonary capillary wedge pressure (PCWP). Bounding pulses, hepatosplenomegaly, and pitting edema would be present in the patient with right ventricular failure.

60. The most common complication of an MI is: A. Dysrhythmia B. Congestive heart failure C. Cardiogenic shock D. Pulmonary embolism

Correct answer - A Dysrhythmias are the most common complication of an acute MI. Nearly all patients with acute MI experience transient rhythm alterations. Congestive heart failure is the next most common complication, usually occurring if the heart's pumping performance is impaired. Cardiogenic shock, the most serious complication, occurs in about 15% of patients hospitalized with an acute infarction. Pulmonary embolism results from formation of mural thrombi on a hypo kinetic myocardium or as a result of bed rest (from peripheral thrombus). It may be a more frequent complication than previously I bought because it can go clinically undetected, and its signs and symptoms mimic those of the MI.

29. Which of the following hemodynamic measurements is taken with the balloon inflated? A. Pulmonary capillary wedge pressure (PCWP) B. Cardiac output................................................ C. Pulmonary artery pressure (PAP).................. D. Mixed venous 0 2 saturation.........................

Correct answer - A Measurement of pulmonary capillary wedge pressure (PCWP) is taken with the balloon inflated. This occludes right ventricular pressures, allowing the catheter to read pulmonary venous pressures, which indirectly-measure left ventricular pressures. The balloon is deflated when measuring pulmonary artery pressures or cardiac output and when withdrawing blood to evaluate mixed venous 02 saturation.

39. Preload may be described as follows: A. The stretch produced within the myocardium at the end of diastole B. The volume in ventricles during systole C. The back-up of pressure in the systemic circulation D. The amount of blood returning to the heart

Correct answer - A Preload may be described best as the stretch produced within the myocardium at the end of ventricular diastole. According to Starling's law, the more a myocardial fiber stretches in diastole, the more it contracts in systole. The volume in ventricles after diastole, called the end-diastolic volume, reflects the heart's contractile ability. The back-up of pressure in the systemic circulation, called systemic vascular resistance, refers to afterload. The amount of blood returning to the heart, called venous return, is a prime factor affecting preload.

65. Which factor would not place Mr. L. at risk for left ventricular failure? A. Pulmonary hypertension.... B. Cardiomyopathy................. C. Aortic stenosis..................... D. Acute myocardial infarction

Correct answer - A Pulmonary hypertension results in an enlarged right ventricle, which may progress to right ventricular failure. Cardiomyopathy, aortic stenosis, and acute myocardial infarction would predispose a patient to left ventricular failure.

36. Which of the following results in an elevated pulmonary artery pressure and a normal wedge pressure? A. Pulmonary hypertension B. Pulmonary edema......... C. Left ventricular failure... D.Constrictive pericarditis...

Correct answer - A Pulmonary hypertension would result in an elevated pulmonary artery pressure and a normal wedge pressure. In pulmonary edema and left ventricular failure, both pressures would be elevated. In constrictive pericarditis, the entire heart is compressed, elevating all pressures.

25. If the transducer level is lower than the phlebostatic axis during pulmonary arterial pressure monitoring: A. The readings will be falsely high................. B. An "overwedge" waveform will appear........ C. The curve will be dampened........................ D. Catheter "fling" will occur on the waveform

Correct answer - A Readings will be falsely high if the transducer level is lower than the phlebostatic axis during pulmonary artery pressure monitoring. This is caused by the additional pressure, or weight, of the fluid within the catheter and tubing, also known as hydrostatic pressure. When the phlebostatic axis and transducer level are at the same height, effective of hydrostatic pressure are negated. Catheter fling will occur on the waveform when the tip of the balloon flotation catheter is close to the pulmonic valve. The turbulence caused by the valve opening and closing results in a chaotic, spiking waveform known as a catheter fling.

90. Which of the following would not precipitate an anginal episode? A. Hypokalemia B. Physical activity C. Smoking D. Heavy meals

Correct answer - A The precipitating factors for angina pectoris are physical activity (not necessarily strenuous), emotional excitement, smoking, or heavy meals. Nocturnal angina (pain when the patient is in a recumbent position) may also be a precipitating factor.

53. The isoenzymes most specific for cardiac muscle damage are: A. CPK-MB and LDH1 B. CPK-MM and LDH3 C. CPK-MM and LDH1 D. CPK-MB and LDH3

Correct answer - A The isoenzymes most specific for cardiac muscle damage are CPK-MB and LDH1 . The heart is the only organ that contains more than a trace amount of CPK-MB. CPK-MB is undetectable in the absence of myocardial damage (so isoenzyme assays will read 100% CPK-MB), but levels can reach 15% if myocardial necrosis occurs. LDH1 is released during myocardial damage but also is found in other organs - such as the kidneys and stomach - and in erythrocytes. Although not as organ-specific as CPK-MB, LDH1 has a delayed peak, so its measurement is useful in patients presenting to the emergency room more than 48 hours after onset of symptoms.

43. Occlusion of the left anterior descending coronary artery may produce all of the following except: A. Massive left ventricular infarction B. Anterior infarction C. Septal infarction D. Right bundle branch block

Correct answer - A The left coronary artery branches into the left anterior descending artery, which supplies the anterior part of the interventricular septum, the anterior wall of the left ventricle, the right bundle branch, and the anterior superior division of the left bundle branch. Thus, occlusion of the left anterior descending artery may result in an anterior wall infarction, a septal infarction, and a right bundle branch block.

61. The nurse could expect Mr.I. to develop all of the following complications except: A. Mobitz II heart block... B. Sinus bradycardia...... C. Sick sinus syndrome.. D. Idioventricular rhythm

Correct answer - A The nurse could expect the patient with an inferior wall MI to develop sinus bradycardia, sick sinus syndrome, or idioventricular rhythm. These results from ischemia to the sinus and atrioventricular nodes, as the right coronary artery supplies these structures and the inferior wall of the left ventricle. Mobitz II heart block would most likely result as a complication in the patient with an anterior wall MI.

47. Which of the following definitions of temporary pacemaker components is incorrect? A. Proximal electrode: paces the myocardium B. Pacemaker catheter: contains two electrodes C. Output control: regulates energy output D. Rate control: regulates frequency of pacing

Correct answer - A The pacemaker catheter contains two electrodes: the proximal (which senses the patient's inherent rhythm) and the distal (which paces the myocardium). The output control regulates energy output delivered to the distal electrode, measured in milliamperes. The rate control regulates pacing frequency, measured in beats per minute.

26. The proximal lumen of a pulmonary artery catheter inserted in Mr. S. is correctly positioned when it is in the: A. Right atrium............. B. Right ventricle.......... C. Pulmonary artery..... D. Pulmonary capillary

Correct answer - A The proximal lumen of a pulmonary artery catheter is correctly positioned when it is in the right atrium. This lumen is used for fluid administration, blood sampling, and monitoring of right atrial pressure, which is needed to calculate systemic vascular resistance.

165. The above situation is a result of: A. Papillary muscle rupture B. Ventricular septal defect C. Cardiogenic shock D. Right ventricular failure

Correct answer - A A large V wave in the setting of an acute lateral wall MI results from mitral regurgitation as a result of papillary muscle ischemia or rupture.

149. Hyperkalemia may be identified electrocardiographically by all of the following except: A. Prominent U wave B. Peaked T wave C. Widened QRS complex D. Diminished P wave amplitude

Correct answer - A A prominent U wave following a T wave occurs in patients who are hypokalemic. Peaked T waves, widened QRS complexes, and diminished or lost P waves signal hyperkalemia.

72. In a patient with an acute myocardial infarction, an early assessment finding that may indicate cardiogenic shock is: A. s3 heart sound B. Bilateral crackles heard throughout inspiration C. Warm, diaphoretic skin D. Urine output of 30 ml in 1 hour

Correct answer - A An early assessment finding that may indicate cardiogenic shock in the patient with an acute MI is an S3 heart sound. The patient with an acute MI is at high risk of developing cardiogenic shock because of a decrease in the contractile function of the left ventricle. Thus, early recognition of signs and symptoms of left ventricular failure is vital. Initially, the failing ventricle causes decreased cardiac output. The body compensates by increasing sympathetic stimulation to increase the heart rate and to shunt blood to vital organs via peripheral vasoconstriction. The skin is cool and clammy. Tachycardia and an increased afterload from vasoconstriction increase the cardiac work load and oxygen consumption. Decreased ventricular emptying during systole occurs, preload increases, and an S3 may be heard. Pulmonary congestion then follows from the backward transmission of left ventricular pressure, resulting in dyspnea, orthopnea, and bilateral crackles. As left ventricular failure progresses, pulmonary congestion increases and cardiogenic shock ensues.

105. Indications for CABG for Mr.S. would include all of the following except: A. New transmural myocardial infarction B. Chronic disabling angina pectoris C. Unstable angina unresponsive to medical therapy D. Occlusion of the left main coronary artery

Correct answer - A Coronary artery bypass grafting (CABG) is most commonly recommended for patients with unstable, preinfarction, or chronic disabling angina pectoris unresponsive to medical therapy; a hemodynamically significant lesion of the left main coronary artery; three-vessel coronary artery disease; and continuing angina pectoris after myocardiaI infarction. Controversy persists about operating on a patient with a new transmural myocardial infarction. Experience has shown that, for a large transmural infarction, surgery is safer after the infarction heals.

75. Another beneficial effect of vasodilator therapy that Mrs. C. should be assessed for is: A. Decreased pulmonary capillary wedge pressure B. Increased pulmonary artery pressure C. Increased systemic vascular resistance D. Decreased cardiac output

Correct answer - A Decreased pulmonary capillary wedge pressure (PCWP) indicates that the vasodilator is having a positive effect. Reduced left ventricular end diastolic pressure, as evidenced by decreased PCWP, results from increased cardiac output and decreased venous return to the heart.

96. In assessing Mr.K, the nurse would observe: A. Narrowing pulse pressure B. Decreased wedge pressure C. Decreased central venous pressure D. Decreased pulse

Correct answer - A In assessing a patient with cardiac tamponade, the nurse would observe a narrowed pulse pressure. When fluid accumulates in the pericardium, the heart cannot fill completely during diastole. As a result, diastolic pressure rises, and the patient has elevated left (PCWP) and right (CVP) ventricular pressures. The pulse increases to maintain cardiac output.

70. In cardiogenic shock, which physiologic compensatory mechanisms occur to increase blood pressure and cardiac output? A. Increasing heart rate and increasing force of contraction B. Increasing heart rate and decreasing preload C. Decreasing preload and decreasing afterload D. Decreasing heart rate and decreasing preload

Correct answer - A In cardiogenic shock, the sympathetic nervous system is activated physiologically to compensate for the failing heart. Catecholamines, such as epinephrine, increase heart rate and force of contraction in an attempt to maintain cardiac output and blood pressure. These mechanisms are doomed to fail because they increase the workload of the heart, which cannot take the added stress.

160. Which of the following symptoms would support the assessment of anxiety? A. Increased urinary frequency B. Bradycardia C. Muscle weakness D. Flushed Skin

Correct answer - A Increased urinary frequency is associated with anxiety, which occurs when a perceived physical or psychological threat produces symptoms similar to fear. The autonomic nervous system is stimulated to protect oneself from the threat. Visceral reflexes, such as bladder micturition, are stimulated, and blood is shunted away from the brain, GI tract, skin and kidneys to allow the heart and muscles to work more effectively to protect the self Associated signs and symptoms of unreality, faintness, anorexia, cool and clammy skin, and an overall decrease in urine output occur with artery catheter placement. anxiety as do tach- yocardia and increased muscle tension. Although the overall urine output is decreased, a smaller bladder volume is needed to stimulate micturition.

135. The recommended dosages of lidocaine should be decreased by 50% in all of the following conditions except: A. Confusion B. Congestive heart failure C. Shock D. Hepatic disease

Correct answer - A Lidocaine is metabolized through the liver. Congestive heart failure, shock, and hepatic disease impair liver function. Thus, lidocaine dosages should be decreased by 50% in these disorders. Confusion is a toxic effect of lidocaine; its appearance should cause termination of the drug.

168. His blood pressure drops two hours after admission, although his lungs remain clear to auscultation. He has positive JVD, which increases on inspiration. It would be most appropriate at this time to: A. Obtain a right precordial lead tracing B. Continuously monitor in lead III C. Continuously monitor in lead V1 D. Obtain a direct posterior lead tracing

Correct answer - A Mr. L is demonstrating signs of isolated right ventricular failure, most likely as a result of right ventricular myocardial infarction, which occurs in about 1/3 of all inferior wall Mis. Obtaining a right precordial lead electrocardiogram would help to support the diagnosis, since right precordial lead changes disappear early, within about 10 hrs after the onset of chest pain. Typical changes of ST elevation usually appear in leads V4R-V6R. ventricular tachycardia with verapamil may result in severe hypotension and ventricular fibrillation. Since lidocaine enhances A V conduction, using it in the setting of supraventricular tachycardia may increase the ventricular rate.

73. One of the earliest clinical signs of cardiogenic shock is : A. Tachycardia B. Cyanosis C. Fluffy-white chest X-ray D. Hypotension

Correct answer - A One of the earliest signs of cardiogenic shock is tachycardia, as the sympathetic nervous system discharges to compensate for the cardiac insult. Cyanosis is a relatively late sign of shock and rather undependable. A fluffy-white chest X-ray and hypotension occur as the compensatory mechanisms fail to maintain cardiac output.

173. In creating a teaching plan for Mr. L., the nurse realizes he is more likely to comply with a medical regimen if: A. His family expects him to comply B. He values the regimen C. He intends to comply D. He is informed about the regimen

Correct answer - A Patients whose families expect them to comply with a medical regimen are most likely to do so. One study of cardiac patients to determine compliance with diet, stress reduction, activity, and medication found that patients believed their family 's expectations were higher than their own. Although patient values, intention, and knowledge are important, including the family in discharge teaching and rehabilitation can promote compliance and increase self-care.

94. The nurse may expect a patient with Prinzmetal' s angina to expenence: A. Chest pain at rest B. Chest pain always relieved by nitrates C. No ECG changes during the episode D. Cyanotic episodes

Correct answer - A Prinzmetal's angina, also called atypical angina, is characterized electrocardiographically by ST elevation during the episode. Pain is unrelated to exertion and usually occurs at rest.

155. The nurse may detect pulsus magnus in a patient with: A. Thyrotoxicosis B. Cardiac tamponade C. Aortic stenosis 0. Congestive heart failure

Correct answer - A Pulsus magnus is a strong, bounding pulse that results from an increased metabolic rate caused by thyrotoxicosis. Pulsus paradoxus, in which the pulse is absent during inspiration, occurs in cardiac tamponade. Pulsus parvus, a small, weak pulse, may be seen in aortic stenosis. Pulsus alternans, in which every other beat is weak, Is characteristic of congestive heart failure.

118. At the time of Mr. M.' s admission, assessment findings most indicative of hypovolemic shock include: A. Pulsus paradoxus B. A hemoglobin value of 10/dl, hematocrit value of 38% C. A widening pulse pressure D. A central venous pressure (CVP) above 4 mm Hg

Correct answer - A Pulsus paradoxus may be present in patients with hypovolemic shock. Its cardinal sign is an abnormal fall (greater than 10 mm Hg) in systolic blood pressure during inspiration. The systolic sound disappears during inspiration because of pulmonary vascular pooling caused by increased lung expansion and increased intrathoracic negative pressure. When intravascular depletion is present, as in hypovolemic shock, the fall of systolic block pressure during inspiration is exaggerated when venous return to the heart is compromised. The patient may also present with a narrowing pulse pressure, which indicates a fall in cardiac output, and a cardiovascular pressure (CVP) reading below 4, which indicates intravascular depletion. Any single hematocrit or hemoglobin value is an unreliable indicator of the patient's condition. Either an increase or decrease in hemoglobin values may signal hypovolemia; dropping values suggest whole blood loss, whereas rising values suggest loss of plasma. Acute blood loss may cause a significant drop in hemoglobin over 1 to 2 hours; however, in older patients, acute blood loss may not cause a significant drop for at least 12 hours.

91. ST depression indicates: A. Ischemia B. Injury C. Necrosis D. Pericarditis

Correct answer - A ST depression indicates myocardial ischemia. Injury to the myocardium is evidenced electrocardiographically by an ST elevation. Myocardial necrosis is evidenced by a Q wave greater in width than 0.04 mm in two or more leads. Pericarditis presentswith elevated ST segments throughout the chest leads.

150. A common complication of Wolff-Parkinson-White syndrome may include all the following except: A. Third-degree AV block B. Atri al fibrillation C. Ventricular fibrillation 0 . Paroxysmal atrial tachycardia

Correct answer - A WoIff-Parkinson-White (WPW) syndrome results from an abnormal AV pathway between the atria and the ventricles. This syndrome leads to an accelerated conduction of the atrial impulses to the ventri cles, giving rise to recurrent bouts of paroxysmal atrial tachycardia. WPW syndrome presents with a normal P wave, a shortened PR interval, and a widened QRS complex and often mimics ventricular tachycardia. Atrial fibrillation is also common with a rapid ventricular response because the delay at the AV mode is bypassed. This rapid dysrhythmia may deteriorate to ventricular fibrillation if not treated promptly.

45. Which complication would be least likely to occur in the patient with an acute anterior wall myocardial infarction? A. Mobitz II heart block B. Mitral insufficiency C. Right bundle branch block D. Ventricular aneurysm

Correct answer - B A complication least likely to occur in a patient with an anterior wall MI would be mitral insufficiency- a systolic murmur most likely seen in an inferior wall MI patient. A Mobitz II heart block, right bundle branch block (RBBB), and ventricular aneurysm would most likely develop in the patient with an anterior wall MI.

38. Cardiac index is calculated by dividing the cardiac output by the: A. Mean arterial pressure.................... B. Patient's body surface area............. C. Patient's weight................................ D. Pulmonary capillary wedge pressure

Correct answer - B Cardiac index is calculated by dividing the cardiac output by the patient's body surface area, measured in meters squared (m2); because the index relates cardiac output to body size, it reflects body requirements more accurately than cardiac output does. Normal cardiac index is 2.5 to 3.5 liters/minute/m2 body surface area charts can be found in most cardiac textbooks.

30. Flotation of a pulmonary catheter into a wedge (PCWP) position increases the risk of: A. Dysrhythmias........... B. Pulmonary infarction C. Pneumothorax........ D. Infection.................

Correct answer - B Flotation of a pulmonary catheter into the wedge position increases the risk of pulmonary infarction. Although deflated, the catheter still occludes blood flow through the capillary, causing small areas of infarction.

49. What is the first action the nurse should take if Mr. A's temporary pacemaker is not sensing? A. Position the patient on his left side B. Check the sensitivity control C. Turn up the energy output D. Turn off the pacemaker

Correct answer - B If the patient's temporary pacemaker is not sensing, the nurse should first check the sensitivity control, a simple yet often over looked intervention. The dial may have inadvertently been turned to fixed rate. If this is not the case and the patient's rhythm is adequate, the nurse should turn off the pacemaker temporarily (to prevent ventricular fibrillation from R-on-T phenomenon) and notify the physician. If the patient's rhythm is inadequate, turning up the energy output or positioning the patient on the left side may help. The electrode may have moved, and increasing the voltage may cause capture. If this is unsuccessful, positioning the patient on his left may help move the catheter back into position. Either intervention is l a temporary measure; the nurse must notify the physician to reposition the catheter.

57. An inferior wall MI most likely results from occlusion of which artery? A. Anterior descending B. Right coronary C. Circumflex D. Left main coronary

Correct answer - B Occlusion of the right coronary artery will most likely result in an inferior wall MI. About 80% of the population has right dominant circulation (the right coronary artery supplies the inferior and posterior walls of the heart, the sinoatrial and atrioventricular nodes, and the right ventricle). The remaining 20% has left dominant circulation (the left circumflex supplies the inferior and posterior wall \s). The circumflex artery supplies the lateral wall of the left ventricle in all persons. The circumflex is on branch of the left main coronary artery; the other branch is the left anterior descending artery, which supplies the anterior wall of the left ventricle and the septum.

46. Which dysrhythmia is the most significant in an acute myocardial infarction? A. Atrial fibrillation B. Premature ventricular contractions C. First degree atrioventricular block D. Premature junctional contraction

Correct answer - B Premature ventricular contractions are the most significant dysrhythmias in an acute myocardial infarction. Irritability of the ischemic tissue is caused by intracellular potassium depletion (causing an elevated extracellular potassium level), inability of the ischemic area to repolarize ( causing electric current flows from the ischemic tissue), and sympathetic discharge in response to myocardial damage. Irritability may cause ectopic firing from the ventricular cells, resulting in ventricular tachycardia or ventricular fibrillation, both lethal dysrhythmias.

34. Which of the following most accurately indicates changes in left ventricular pressure? A. Central venous pressure.................. B. Pulmonary capillary wedge pressure C. Systemic arterial blood pressure...... D. Pulmonary artery pressure...............

Correct answer - B Pulmonary capillary wedge pressure is the most accurate indicator of changes in left ventricular pressure. In the wedge position, right ventricular pressures are occluded and forward pressures in the pulmonary vasculature are measured. These reflect the heart's left ventricular end-diastolic pressure because an open system exists from the tip of the catheter to the left ventricle as the mitral valve is opened during diastole.

52. Which of the following is least helpful in diagnosing an acute myocardial infarction (MI)? A. Patient's history B. Physical examination C. Enzyme studies D. Serial ECGs

Correct answer - B The physical examination is least helpful in diagnosing an acute myocardial infarction (MI). Physical findings - such as gallops, murmurs, crackles, or hypotension - rarely occur during the early stages of an MI and can indicate various diseases. Any two of three factors - history of MI, positive enzyme studies, or positive ECG - confirm the diagnosis.

27. In which area of the heart would the pulmonary artery catheter be most likely to cause dysrhythmias? A. Right atrium........... B. Right ventricle........ C. Pulmonary artery.... D. Pulmonary capillary

Correct answer - B The pulmonary artery catheter would most likely cause dysrhythmias in the right ventricle. The physician should be notified when a right ventricular waveform is identified so that the catheter can be repositioned promptly.

124. Which is not an indication for thrombolytic therapy? A. An occluded arteriovenous fistula B. Subendocardial myocardial infarction (MI) C. Pulmonary embolus D. Peripheral arterial occlusion

Correct answer - B A subendocardial myocardial infarction (MI), typically a result or decreased perfusion from coronary artery spasm or hypotension, is not an indication for thrombolytic therapy. Subendocardial Mls usually do not occur with thrombotic occlusions. Indications for thrombolytic therapy include a transmural MI, an occluded arterio-venous fistula, pulmonary embolus, and peripheral arterial and venous occlusions.

111. The rationale for inserting a temporary pacemaker during CABG is to: A. Treat trifascicular block, which commonly occurs B. Optimize cardiac output C. Override tachydysrhythmias D. Test stress by rapid atrial pacing

Correct answer - B A temporary pacemaker is inserted during CABG to optimize cardiac output through manipulation of the heart rate. Since heart rate x stroke volume equals cardiac output, increasing the heart rate will optimize cardiac output of the hypodynamic post-operative left ventricle.

186. A frequent postoperative cardiac transplant nursing diagnosis Mrs. R. should be assessed for is: A. Powerlessness B. Body image disturbance C. Ineffective denial D. Hopelessness

Correct answer - B After a cardiac transplant, the problem of body image disturbance may appear. Not only due to the presence of an external scar, this problem may also result from the removal of such a vital organ as a heart along with receiving someone else's heart. Body image disturbance may cause the patient to withdraw and hamper normal coping mechanisms. Allowing the patient to discuss her feelings, while accepting her nonjudgementally, may foster her acceptance of the change.

112. All postoperative cardiac patients develop: A. Confusion B. Pericarditis c. Hypokal emia 0. Atelectasis

Correct answer - B All postoperative cardiac patients develop pericarditis because of the pericardiotomy, which causes inflammation. Confusion, hypokalemia, and atelectasis may also result from surgery, but these can be prevented with careful treatment.

93. The nurse auscultates an S4 gallop during her assessment of Mr.A. The appearance of an S4 gallop during an anginal episode signifies: A. Congestive heart failure B. Decreased compliance of ischemic zone C. Aortic stenosis D. Increased left ventricular filling volume

Correct answer - B Although an S 4 results from an increase in left ventricular filling volume and is often heard in patients with congestive heart failure and aortic stenosis, an S4 during an anginal episode signifies decreased compliance of the ischemic zone.

159. The earliest clue in assessing for a dissecting aortic aneurysm would most likely be: A. Diaphoresis B. Back pain C. Dependent edema D. Crackles

Correct answer - B Back pain may be an early clue when assessing for a dissecting aortic aneurysm. The pain may signal the onset of a dissection. Caused by blood leakage into the intima, the pain usually is described as excruciating and tearing an may migrate as the dissection progresses.

172. During his first few days of hopitalization, Mr. L. exhibits signs of deniaL At this time, patients with a myocardial infarction may experience a positive correlation between denial and; A. More complications B. A better survival rate C. An increase in mortality D. More psychological problems

Correct answer - B During the first days of hospitalization for acute myocardial infarction, a patient may experience a positive correlation between denial and a better survival rate. A defense mechanism that inhibits anxiety, denial may protect against heart stimulation that anxiety can produce. However, denial in the recovery phase - when the patient may be noncompliant and does not recognize signs of further cardiac events -may decrease survival probability.

122. Fluid replacement in the hypovolemic patient is best gauged by which parameter? A. Blood pressure B. Pulmonary capillary wedge pressure C. Systemic vascular resistance D. Pulmonary artery pressure

Correct answer - B Hemodynamic measurements obtained by pulmonary arterial catheters provide valuable information. Fluid replacement is monitored best by the pulmonary capillary wedge pressure (PCWP). If PCWP is low, the patient needs fluids. The systemic vascular resistance provides information for vasodilator or vasoconstrictor therapy Pulmonary artery pressure approximates venous pressure within the lungs and may increase in pulmonary hypertension or hypoxia, regardless of the patient's fluid status.

157. Hypertension causes a reflex bradycardia from stimulation of: A. Adrenergic receptors B. Baroreceptors C. Alpha receptors D. Beta receptors

Correct answer - B Hypertension causes a reflex bradycardia from stimulation of the baroreceptors. Increases in arterial pressure stimulate the vasopressor area of the vasomotor center, which then increases vagal tone and slows heart rate. Adrenergic receptors refer to postganglionic sympathetic fibers, which secrete norepinephrine. Alpha-and beta-adrenergic receptors, when stimulated, cause the heart rate to accelerate. Alpha receptors also cause vasoconstriction, whereas beta receptor stimulation results in vasodilation and a stronger cardiac contraction.

138. During the code, Mr. Q.' s monitor shows sinus bradycardia, but no pulse is present. Pulseless electrical activity (PEA) is diagnosed. Initial treatment should be to administer: A. Calcium chloride B. Epinephrine C. Atropine D. Sodium bicarbonate

Correct answer - B If sinus bradycardia is evident on the monitor but no pulse is present, the patient is in pulseless electrical activity (PEA). The first priority is to resume CPR. Epinephrine 1 mg should be the first medication given, either intravenously or by endotracheal tube. This amount should be repeated at least every 5 minutes.

80. Which is the first parameter to respond to intravenous Lasix? A. Urine output B. PCWP C. Breath sounds D. Chest X-rays

Correct answer - B Lasix has an initial immediate vasodilatory effect, causing venous pooling, decreasing pulmonary venous congestion, and decreasing pulmonary capillary wedge pressure. Lasix also has a later diuretic effect, reflected in an increase in urine output. Breath sounds and chest X-rays may take hours to improve.

185. Mrs. R. is started on cyclosporine (Sandimmune) postoperatively. In assessing for complications related to this drug therapy, the nurse should monitor Mrs. R. 's A. Blood glucose B. Serum creatinine C. Serum amylase D. Serum magnesium

Correct answer - B Nephrotoxicity and hepatotoxicity are the two major side effects of cyclosporine therapy. Monitor for a rise in serum creatinine and liver enzymes.

166. After the procedure, Mr. V.'s rhythm changes into a wide QRS tachycardia. The physician is unable to deterrmine whether the rhythm is of venticular or atrial origin. In situations like this, it is best to administer: A. Lidocaine B. Procainamide C. Verapamil D. Adenosine

Correct answer - B Since procainamide has both atrial and ventricular antiarrhythmic properties, it should be used in the setting of a wide QRS tachycardia in which the origin of the arrhythmia is unknown. Treating ventriculartachycardia with verapamil may result in severe hypotension and ventricular fibrillation. Since lidocaine enhances A V conduction, using it in the setting of supraventricular tachycardia may increase the ventricular rate.

163. The reason to monitor in this lead is to observe for which complication? A. Ventricular dysrhythmias B. RBBB C. LBBB D. Septal perforation

Correct answer - B Since the right bundle branch lies close to the endocardial surface of the right ventricle, it may easily be traumatized during pulmonary artery catheter placement.

84. The therapeutic effects of the IABP include all of the following except: A. Increased cardiac output B. Increased cardiac afterload C. Decreased myocardial 0 2 consumption D. Increased aortic pressure

Correct answer - B The therapeutic effect of the IABP is to increase cardiac output, decrease myocardial 0 2 consumption, increase aortic pressure, and decrease cardiac afterload. The IABP, when deflated during systole, displaces blood volume in the aorta, which lowers aortic end-diastolic pressure and resistance to left ventricular ejection, decreasing afterload. Displacement of the blood volume allows more efficient pumping of the left ventricle, increases cardiac output, and decreases myocardial 0 2 consumption. This preserves the already compr0mised myocardium.

175. Which electrocardiographic parameter should be monitored with quinidine therapy? A. PR interval B. Electrical axis C. QT interval D. Heart rate

Correct answer - B Torsadc de pointes, or polymorphous ventricular tachycardia, has a twisting characteristic to the QRS complexes. This rhythm usually results from a prolonged QT interval. It is important to differentiate this dysthmia from ventricular tachycardia or fibrillation because conventional therapy is usually ineffective.

167. During the admission history, Mr. L. tells the nurse that he has been told that he is a "Type A" personality. Which comparative statement about Type A and B behaviors during hospitalization foran acute myocardial infarction is true? A. Type B behavior uses denial as a defense mechanism more often B. Type A behavior is associated with shorter hospitalization C. Type B personalities are more concerned with death D. Type B personalities tend to exhibit more Type A behaviors during hospitalization

Correct answer - B Type A behavior, associated with shorter hospital stays than Type B behavior, is characterized by time urgency, impatience, and aggressiveness; Type B personalities are more relaxed and introverted. Type A personalities are twice as likely to develop heart disease. During acute myocardial infarction, Type A personalities tend to deny illness and don't want to waste time in the hospital, especially if they don't feel sick. Therefore, while they tend to have shorter hospital stays, they run a greater risk of fatal complications resulting from their denial.

170. Correct therapy at this time would include: A. Intraaortic balloon pump B. Volume expansion C. Vasodilator therapy D. All of the above

Correct answer - B Volume expansion is indicated in the setting of right ventricular MI to increase forward flow to the left side of the heart, and to increase wedge pressure, cardiac output and blood pressure.

137. Mr.Q. 's rhythm progresses to ventricular fibrillation. After three defibrillation attempts, which medication should be administered first? A. Sodium bicarbonate B. Epinephrine 1 mg LV. C. Lidocaine I 00 mg I. V. bolus D. Atropine 0.5 mg LV.

Correct answer - B When a patient is in ventricular fibrillation and three defibrillation attempts have been made at 200, 300 and 360 joules, epinephrine 1 mg should be administered intravenously or via endotracheal tube.

64. Hepatojugular reflux suggests: A. Chronic liver condition B. Obstructive lung disease C. Congestive heart failure D. Acute liver dysfunction

Correct answer - C A hepatojugular reflux is elicited by exerting firm and sustained pressure with your hand over the patient's right upper quadrant for 30 to 60 seconds. Watch for increased jugular venous pressure during this maneuver. A rise of more than 1 cm is abnormal (a positive hepatojugular reflux) and suggests congestive heart failure.

54. In evaluating LDH isoenzymes, which report would indicate myocardial damage? A. LDH5 greater than LDH4 B. LDH2 greater than LDH1 C. LDH1 greater than LDH2 D. LDH4 greater than LDH5

Correct answer - C A report of LDH, greater than LDH2 indicates myocardial damage. Normally, LDH2 is greater than LDH1. If an LDH1 rich organ other than the heart is damaged, LDH2 usually rises. Damage to the heart or erythrocytes would not cause a rise in LDH2 . Therefore, to prevent erythrocyte damage, one should not hemolyze an LDH isoenzyme spectmen.

32. If the pulmonary artery catheter in Mr. S. was reading 30/14 and then you noticed the continuous reading of 30/4, you would suspect: A. Decreased diastolic pressure due to beneficial effect of medications B. Decreased pulmonary artery pressure due to increased anoxia C. Catheter moved into the right ventricle D. Catheter advanced distally

Correct answer - C A sudden drop in the diastolic pressure during hemodynamic monitoring indicates the catheter has moved back to the right ventricle. Normally, right ventricular and pulmonary artery systolic pressures are equal. However, during diastole, the pulmonic valve closes, separating the right ventricle from the pulmonary artery. Right ventricular diastolic pressure, then, is lower (normally 0 to 5 mm Hg) because it reflects right atrial pressure, a low pressure chamber. Pulmonary artery diastolic pressure is higher (normally 8 to 12 mm Hg) because it reflects left ventricular pressure, a higher pressure chamber.

40. Afterload may be described as follows: A. The measurement of the left ventricular end-diastolic pressure B. The amount of force of ventricular contraction C. The impedance of ejection of blood from the left ventricle D. The amount of blood remaining in the left ventricle after systole

Correct answer - C Afterload may be described best as the impedance to ejection of blood from the left ventricle. This is clinically represented by the mean arterial pressure and systemic vascular resistance. The measurement of the left ventricular end-diastolic pressure is the clinical representation of preload and is indirectly determined by the pulmonary capillary wedge pressure. The amount of force of ventricular contraction is called contractility. The amount of blood remaining in the left ventricle after systole is called the left ventricular end-systolic volume, or residual volume.

37. Mr.C., age 48, was admitted with increasing dyspnea and swollen feet. Upon Swan-Ganz catheterization, his pressures were as follows : right atrial=20, right ventricular=70/22, pulmonary artery=70/36, and pulmonary capillary wedge=12. His condition would be diagnosed as: A. Congestive heart failure B. Cardiac tamponade...... C. Pulmonary hypertension D. Hypovolemia.................

Correct answer - C An elevated right ventricular pressure and a normal wedge pressure indicate pulmonary hypertension. Increasing dyspnea and swollen feet also suggest right ventricular failure. In congestive heart failure, the wedge pressure would be high, although right ventricular pressure may not be high initially. In cardiac tamponade, pericardia! fluid would elevate all pressures within the heart. Pressures would be low in hypovolemia.

55. The enzyme that returns to normal level 7 to 10 days after myocardial damage is: A. SGPT B. CPK C. LDH D. SGOT

Correct answer - C LDH returns to normal 7 to 10 days after an infarction. The onset of LDH elevation occurs 12 to 24 hours after tissue damage and peaks at 72 hours. CPK begins to rise 4 to 6 hours after infarction, with a peak by 12 to 20 hours, and usually returns to normal by the 2nd or 3rd day. SGOT starts to increase in about 6 to 8 hours, peaks in 18 to 36 hours, and tapers off about the 4th to 6th day. SGPT is more specific for liver disease but may rise if the myocar-dial damage is accompanied by shock or severe congestive heart failure that results in liver congestion or damage.

28. Which of the following interventions would be used first to correct dysrhythmias associated with catheter insertion? A. Administration of lidocaine B. Defibrillation..................... C. Withdrawal of catheter tip D. Cardioversion...................

Correct answer - C Withdrawing the catheter tip from the right ventricle would be the first and simplest intervention to correct a dysrhythmia caused by catheter insertion. If unsuccessful, then defibrillation, cardioversion, or lidocaine administration could be used to terminate the dysrhythmia.

107. Which condition contraindicates CABG? A. Recent angioplasty B. Subenclocarclialmyocardial infarction C. Preoperative ejection fraction of 10% without angina D. Malignant dysrhythmias

Correct answer - C An ejection fraction of 10% in the absence of angina is a contraindication to CABG because the patient has diminished myocardial function preoperatively and would have a poor prognosis. If angina is present, the ejection fraction may be low because of hypoperfusion to the myocardium. The ejection fraction may improve when the pain is relieved.

134. During prophylactic administration of lidocaine, after what time interval should the second bolus be given to Mr.Q.? A. 2 minutes B. 5 minutes C. 10 minutes D. 15 minutes

Correct answer - C Because the half-life oflidocaine's distribution phase is 8 minutes, the recommendation is to provide a second bolus after 10 minutes. This prevents a rapid decline of the plasma level and avoids toxic effects from rapid administration. However, for terminating dysrhythmias - rather than for prophylactic use- it may be necessary to give a second bolus 1 to 2 minutes after the first.

100. An emergency pericardiocentesis is performed on Mr.K. All of the following are common complications of pericardiocentesis except: A. Ventricular perforation B. Pneumothorax C. Complete heart block D. Ventricular fibrillation

Correct answer - C Heart block is not a complication of pericardiocentesis. Ventricular perforation can occur if the needle is advanced too far and ventricular fibrillation can result if the needle contacts the myocardium . Pneumothorax can occur if the needle is accidentally introduced into the pleural space.

85. A nursing measure that would be inappropriate while the IABP is inserted would be to: A. Perform passive range-of-motion exercises four times a day B. Monitor pulses frequently C. Keep the head of the bed elevated 45 to 60 degrees D. Administer antipyretics for temperature above 101 OF.

Correct answer - C It would be inappropriate to elevate the head of the bed 45 to 60 degrees, as this may increase flexion of the hip- thereby occluding or kinking the IABP catheter at the insertion into the femoral artery. The head of the bed must be elevated no higher than 45 degrees. Performing passive range-of-motion (ROM) exercises, monitoring pulses and administering antipyretics for a temperature above 101°F. are appropriate actions. Remember to use salicylates with caution because the patient is anticoagulated while on the IABP.

119. Patient assessment data that suggest hypovolemia as a cause of shock include: A. Decreased central venous pressure and distended neck veins only while the patient is supine B. Increased urine sodium level and a urine specific gravity of 1.010 C. Decreased pulse pressure D. Tachycardia with ventricular gallop (S3)

Correct answer - C Patient assessment data suggesting hypovolemia as the cause of shock include an abnormal tilt test. This assessment tool indicates circulating blood volume loss by measuring blood pressure and pulse rate changes during patient position changes. Abnormal tilt test findings include improved blood pressure when the patient's legs are raised above heart level , or a I0 mm Hg or greater drop in systolic blood pressure between supine and sitting positions. Pulse pressure is a far better indicator of early shock than is blood pressure. Pulse pressure is related to cardiac stroke volume, thus, changes provide a good indication ofblood flow. During the early stages of shock, the systolic pressure usually drops faster than the diastolic pressure, thereby narrowing or decreasing the pulse pressure. The hypovolemic patient will have flat neck veins when supine, a low CVP, decreased urine sodium level, and a high urine specific gravity. An S3 is heard when left ventricular end-diastolic pressure (L VEDP) increases - as found in congestive heart failure (CHF) or cardiogenic shock. Tachycardia occurs in all forms of shock, as the heart attempts to increase cardiac output.

174. Ms. N.'s EKG shows deep T wave inversion in leads V2 and V3 but is otherwise normaL This finding is consistent with: A. Posterior wall MI B. Pericarditis C. Anterioseptal MI D. Wellen's syndrome

Correct answer - C Patients on quinidine, a class IA antiarrhythmic, should have their QT interval monitored for prolongation, which may precipitate torsade de pointes. Other class IA antiarrhythmics are procainamide (Procan, Pronestyl) and disopyramide (Norpace).

113. A nursing intervention for a patient experiencing pain from pericarditis would be to: A. Instruct the patient on the importance of bed rest B. Take the patient's blood pressure before giving nitroglycerin for pericardial pain C. Encourage the patient to sit up and lean forward D. Encourage the patient to breathe deeply

Correct answer - C Pericardial pain is typically relieved by encouraging the patient to sit up and lean forward. The pain is usually aggravated by deep inspiration and relieved by anti-inflammatory agents rather than nitroglycerin. These three hallmarks differentiate pericardia! pain from the pain of acute myocardial infarction (MI).

147. Progressive prolongation of the PR interval followed by a blocked P wave describes : A. Sinus arrest B. First-degree A V block C. Second-degree AV block (Mobitz Type I or Wenckebach) D. Second-degree AV block (Mobitz Type II)

Correct answer - C Progressive prolongation of the PR interval followed by a blocked P wave indicates second-degree A V block (Mobitz Type I). This is usually a transient rhythm, signaling a progression of the block from first-degree to third-degree. It occurs in acute myocardial infarction, digitalis toxicity, and degenerative disease of the A V node.

98. Pulsus paradoxus can be defined as: A. An absence of pulse during expiration B. A decrease of 20 mm Hg in arterial systolic pressure on expiration during auscultation of blood pressure C. A drop of more than 10 mm Hg in arterial systolic pressure on inspiration D. A drop of 20 mm Hg in arterial systolic pressure on deep in spiratory effort

Correct answer - C Pulsus paradoxus can be defined as the absence of a pulse or no auscultation of Korotkoff sounds for more than 10 mm Hg in arterial systolic pressure during inspiration. A drop in pressure of less than I 0 mm Hg is a normal response to inspiration. Inspiratory effort causes a rise in intrathoracic pressure, which compresses the great vessels to the heart and diminishes venous return to the heart. Also, inspiration causes pulmonary venous pooling, which further decreases blood return to the left side of the heart and, as a result, decreases stroke volume during inspiration.

82. The treatment least effective in protecting the myocardium from further damage is: A. Decreasing peripheral vascular resistance B. Decreasing ventricular preload C. Increasing left ventricular afterload 0. Increasing cardiac output

Correct answer - C The goals of therapy for the patient in cardiogenic shock are to improve myocardial contractility, increase cardiac output, and reduce the myocardial demand for oxygen. Improvement of cardiovascular function is primarily managed by pharmacologic therapies. Nitroprusside (Nipride) and nitroglycerin (Tridil) are commonly used to decrease both preload (ventricular end-diastolic pressure) and afterload (vascular resistance). These vasodilators reduce blood return to the heart and decrease systemic peripheral vascular resistance. Cardiac output is further enhanced by a decrease in cardiac work load and oxygen demand and by complete emptying of the ventricles during systole. Vasodilator therapy is used with sympathomimetic drugs, such as dopamine (Inotropin) and dobutamine (Dobutrex). These vasopressors have an inotropic effect that enhances myocardial contractility and cardiac output.

156. Which valve is most commonly involved in infective endocarditis? A. Tricuspid B. Pulmonic c. Mitral D. Aortic

Correct answer - C The mitral valve is the most common site for infective endocarditis, followed by the aortic valve. Infection of the tricuspid valve is rare but occurs among I. V. drug abusers. Pulmonic valve endocarditis is the rarest form of endocarditis.

104. Which patient is the least appropriate candidate for angioplasty? The patient with: A. New-onset angina B. Single vessel disease C. Mu~tiple areas of obstruction in a single vessel D. An ejection fraction of 50%

Correct answer - C The patient with multiple areas of obstruction in a single vessel is the least appropriate candidate for angioplasty; less than 10% of patients with coronary artery disease are candidates for the procedure. To maximize success, patients with diffuse cardiac disease are not recommended for angioplasty. An ejection fraction of 50% shows good cardiac function and makes the patient a good candidate for bypass grafting, a possible complication of angioplasty.

102. Which of the following is not an invasive diagnostic technique for cardiac assessment? A. His bundle ECG B. Ventriculography C. Vectorcardiography D. Aortography

Correct answer - C Vectorcardiography is a form of electrocardiography that measures the direction and magnitude of electrical forces. It is more diagnostic than a normal ECG, especially for bundle branch blocks, fascicular blocks, and myocardial infarction. A His bundle ECG involves passing an electrode catheter into the right side of the heart and positioning it near the tricuspid valve. The catheter specifies the site of a conduction delay, such as the atrioventricular node, His bundle, or bundle branch. Ventriculography is a radiographic test that uses a contrast medium to evaluate ventricular function . Aortography is an arteriogram of the aorta used to diagnose aortic aneurysms, coarctation of the aorta, or aortic valve incompetency.

35. A normal wedge pressure, increased pulmonary artery pressures, and evidence of right ventricular failure would most likely indicate: A. Cardiac tamponade B. Left ventricular failure C. Myocardial infarction D. Pulmonary embolism

Correct answer - D A normal wedge pressure, increased pulmonary artery pressures, and signs of right ventricular failure may indicate a pulmonary embolus. Because the pulmonary vasculature is situated between the right and left sides of the heart, respiratory disease would raise right ventricular pressures. Wedge pressure, a reflection of left ventricular pressures, would not be elevated.

56. The inferior wall MI in Mr. I. will show indicative changes in leads: A. V1 to V4 B. V1. aVL C. vs. v6 D. II, III, a VF.

Correct answer - D An inferior wall MI will show indicative changes in leads I, II, III and a VF because these leads look at the inferior wall. Changes in leads V1 to V4 indicate an anterior-septal infarction because these leads look at the front of the heart, most specifically the septum. Changes in VI, a VL, V5 and V6 indicate a lateral wall infarction because these leads look at the heart from the lateral side of the left ventricle.

48. Which is least likely to contribute to the inability of a permanent pacemaker to sense the patient's rhythm? A. Battery failure B. Improper position of catheter C. Faulty sensing mechanism D. Electromagnetic interference

Correct answer - D Electromagnetic interference rarely causes hazards with newer permanent pacemakers. (However, in temporary pacemakers, electrocautery may override the sensing mechanisms.) Battery failure, improper catheter position, a faulty sensing mechanism, and a lead wire fracture can result in sensing failure.

158. Malignant hypertension is characterized by a diastolic blood pressure over: A. 90 mm Hg B. 100 mm Hg C. 110 mm Hg D. 140 mm Hg

Correct answer - D Malignant hypertension is characterized by a diastolic blood pressure greater than 140 mm Hg. Other clinical presentations include retinopathy with hemorrhage and papilledema, impaired renal function, and hypertensive encephalopathy.

97. Which is not characteristic of pericardia! tamponade? A. Rise in central venous pressure B. Pulsus paradoxus C. Distant heart sounds D. Widening pulse pressure

Correct answer - D Pulse pressure narrows because of the rise in diastolic filling pressure. An elevated CVP and pulsus paradoxus are common symptoms of tamponade. Heart sounds may be distant because they can be muffled by pericardia! fluid.

63. Which of the following statements about jugular venous pressure is false? A. Examination of the jugular veins provides important information about cardiac compensation B. Internal jugular pulsations give a more accurate pressure than do external jugular pulsations C. When examining for jugular venous pressure, the nurse must elevate the head of the bed 3 0 degrees D. Pulsations should be absent while the patient is lying flat

Correct answer - D Examination of the jugular veins and their pulsations allows an accurate estimation of central veneus pressure and therefore gives important information about cardiac compensation. The internal jugular pulsations provide a more accurate reading than external jugular pulsations. When examining for jugular venous pressure, the nurse should elevate the head of the bed 30 degrees. When the patient is in a horizontal position, jugular venous pulsations should be present; if they are absent, blood volume depletion may be suspected.

62. Which of the following would be typical of right ventricular failure? A. Pulmonary capillary wedge pressure greater than 25mmHg B. Jugular venous distention 2 em above the clavicle C. Bilateral moist rales D. Hepatosplenomegaly

Correct answer - D Hepatosplenomegaly would reflect right ventricular failure. Other clinical signs include dependent pitting edema,jugularvenous distention greater than 3 em above the sternal angle, and elevated central venous pressure readings. A pulmonary capillary wedge pressure (PCWP) above 25 mm Hg and bilateral moist crackles reflect left ventricular failure.

33. Which of the following pressures are within normal limits? A. PAP=34/24, W=l2 B. PAP=30/20, W=lO C. P AP=28/28, W=20 D. PAP=24/14, W=l2

Correct answer - D Normal pulmonary artery systolic pressure is between 20 and 30 mm Hg; normal pulmonary artery diastolic pressure is between 8 and 14 mm Hg; and normal pulmonary capillary wedge pressure is between 8 and 14 mm Hg. Pulmonary artery diastolic and capillary wedge pressures should be similar in the absence of disease.

115. Which of the following statements is true about cardiopulmonary resuscitation in the immediate postoperative cardiac surgery patient? A. Abdominal instead of sternal thrusts must be used B. Defibrillator may not be set higher than 200 joules C. The chest may be compressed only 1/2 inch D. Open-chest cardiac massage must be performed

Correct answer - D Open-chest cardiac massage must be carried out in CPR for the post-operative cardiac surgery patient. Effective massage consists of compressing the heart between the sternum and spinal cord, increasing intrathoracic pressure. During cardiac surgery, the sternum is broken to gain access to the mediastinum. Therefore, ostoperatively, the sternum is not intact and would not provide effective chest compressions.

Mr. A. has a VVI pacemaker. What does the first initial of this pacemaker indicate? A. The location of the catheter B. Which chamber is sensed C. The mode of the pacemaker D. Which chamber is paced

Correct answer - D Pacemaker types are written as a three-letter code. The first initial refers to the chamber paced, the second to the sensing chamber, and the third to the mode of response. Thus, a VVI pacemaker is one that has ventricular pacing and sensing and inhibited mode (demand). The following initials are used: V =ventricular, A= atrium, D =double chamber, I = inhibited, T =triggered, and 0 = not applicable.

31. Prolonged inflation of the catheter balloon increases the risk of: A. Dysrhythmias............... B. Air embolism................ C. Pneumothorax.............. D. Pulmonary hemorrhage

Correct answer - D Prolonged inflation of the catheter balloon increases in risk of pulmonary hemorrhage. An inflated balloon presses against fragile capillary vessels and may rupture them if inflated too long. Suspect this if hemoptysis appears in the patient.

44. The nurse would anticipate Mr.A.'s 12-lead ECG to show: A. ST elevations in V5 and V6 B. Q waves in II and a VL C. ST elevations in II, III and a VF D. ST elevations in VI and v2

Correct answer - D The nurse would anticipate a 12-leadECGofa patient with an anterior wall MI to show ST elevations in leads V, and V2 with reciprocal ST depressions in leads II, III and a VF.

68. Which of the following hemodynamic parameters would indicate left ventricular failure in a patient with COPD? A. PAP-54/22, PCWP-14 B. PAP-48/26, PCWP-16 C. PAP-22/12, PCWP-16 D. PAP-48/26, PCWP-20

Correct answer - D A pulmonary artery pressure (PAP) of 48/26, a PCWP of 20 would reflect an increase in left ventricular end-diastolic pressure (LVEDP), such as in left ventricular failure.

95. Which of the following statements is false regarding Prinzmetal' s angina? A. It is the result of coronary artery spasm B. Calcium blockers are the pharmacological treatment C. It occurs in 5% to 10% of angina cases D. Surgical intervention is warranted

Correct answer - D About 5% to 10% of angina pectoris cases result from coronary artery spasm. This syndrome, termed Prinzmetal's (variant) angina, is characterized by anginal pain at rest with associated ST segment elevation. Treatment consists of calcium channel-blocking agents. Surgical intervention for coronary artery spasm is not indicated, especially since the decrease in blood supply is due to spasms rather than atherosclerosis.

120. An indication of shock is a drop in systolic blood pressure below: A. 100 mmHg B. 96 mm Hg C. 90 mm Hg D. 80 mmHg

Correct answer - D An indication of shock is a drop in systolic blood pressure below 80 mm Hg. When the compensatory mechanism of vasoconstriction fails to maintain blood pressure, the decompensatory stage sets in and reflects a systolic pressure below 80 mm Hg.

184. Acute rejection in cardiac transplantation is rdiagnosed by: A. ECG B. Chest X-ray c. Echocardiography D. Endomyocardial biopsy

Correct answer - D Endomyocardial biopsy, performed transvenously, remains the only definitive method for documenting acute rejection.

136. The nurse's first response would be to: A. Administer lidocaine 100 mg I. V. bolus B. Defibrillate with 200 joules C. Administer a precordial thump D. Perform immediate synchronized cardioversion with 100 to 200 joules

Correct answer - D If the patient has a pulse but is hemodynamically unstable, immediate cardioversion with 100 to 200 joules must take precedence over antidysrhythmic therapy.

177. The treatment of choice for this dysrhythmia is: A. Adenosine B. Cardioversion c. Defibrillation D. Magnesium

Correct answer - D Magnesium sulfate is the treatment of choice for torsade de pointes. 1-2 gm is administerd over 1 to 2 minutes, followed by a continuous infusion of 0.5-1 gm/hr. Other treatment which may be effective include overdrive pacing and isoproterenol (Isuprel).

114. Pericarditis may be suspected if the patient's ECG shows: A. First-degree A V block B. Prominent U wave c. Prominent R wave D. ST elevations

Correct answer - D ST elevations in two or three of the standard leads and some or all of the precordial leads are characteristic changes in the ECG of a patient with pericarditis. This occurs early in the course, followed by ST segments returning to baseline and T wave flattening, then inversion, in all leads except a VR. The absence of Q waves, the loss of R wave voltage in V leads, and a general distribution of ST segment changes help to differentiate pericarditis from an acute MI.

116. Which of the following phrases best describes shock? A.The inability of the body to excrete metabolic waste products B.The collapse of the respiratory system C. The collapse of the sympathetic nervous system D. A state of inadequate tissue perfusion

Correct answer - D Shock is a syndrome characterized by impaired cellular metabolism from decreased blood flow - a state of inadequate tissue perfusion. The major pathophysiologic problem occurs in the microcirculation at the capillary level. Inadequate tissue perfusion and decreased oxygen delivery result in a constellation of physiologic responses. As shock develops, a fall in cardiac output triggers compensatory mechanisms as pressoreceptors are stimulated to cause vasoconstriction. The net effect is an increase in mean arterial pressure and a restriction of peripheral blood flow. The available circulating volume is shunted to perfuse vital organs. As peripheral vasoconstriction progresses, a marked reduction in oxygen delivery to the cells forces a switch from aerobic to anaerobic metabolism. Increased lactic acid is produced resulting in respiratory compensation; body-acid load is reduced by blowing off carbon dioxide. As a shock state continues, respiratory compensation fails and the progressive decline in pH from excessive lactate levels ensues. Metabolic acidosis causes impaired cellular function and impaired capillary permeability. Intravascular volume is further decreased as blood flows into capillary beds. The patient is now in the progressive state of shock as compensatory mechanisms fail to maintain an adequate cardiac output.

103. Technetium phosphate imaging is performed on Mr.S. This test: A. Uses cold spot imaging B. Determines coronary artery blockage C. Calculates left ventricular ejection fraction D. Specifies the area of myocardial damage

Correct answer - D Technetium phosphate imaging specifies the area of myocardial damage; technetium phosphate accumulates in the area of damage, showing a hot spot. Thallium scintiography uses cold spot imaging to reveal perfusion defects. Coronary arteriography determines coronary artery blockage. A gated pool study, which uses a radionuclide dye, helps evaluate ventricular ejection fraction.

178. Which statement about t-P A and streptokinase is true? A. They are equally effective B. They both activate circulating and fibrin-bound plasminogen c. Only t-P A depletes clotting factors D. The half-life of t-P A is considerably shorter than that of streptokinase

Correct answer - D The half life oft-PAis considerably shorter than that of streptokinase (5 minutes vs. 18 minutes). This is a benefit ifbleeding occurs- the effects of thrombolysis can be quickly reversed. Clinical studies indicate that t-PA is 25% to 50% more effective in thrombolysis than streptokinase. Clot-specific, t-P A only activates fibrin-bound plasminogen. Streptokinase activates circulating plasminogen and can deplete clotting factors, producing a systemic lytic effect.

92. The pathologic changes found on an ECG to indicate myocardial ischemia are: A. ST elevation B. ST segment depression and T wave elevation C. Q wave formation D. ST segment depression and T wave inversion

Correct answer - D The pathologic changes found on an ECG that indicate myocardial ischemia are ST segment depression and T wave inversion. ST elevation indicates injury, except in pericarditis. ST segment depression and T wave elevation in V1 and V2 may indicate posterior wall injury because of the mirror image one obtains when looking for posterior wall damage. Q wave formation is diagnostic of myocardial damage.

74. Vasodilator therapy is instituted for Mrs. C. Vasodilator therapy exerts a therapeutic effect in cardiogenic shock by: A. Decreasing venous return B. Reducing peripheral vascular resistance C. Enhancing cardiac output D. All of the above

Correct answer - D Vasodilator therapy relaxes the systemic vessels, producing venous pooling of blood and decreasing venous return. Less blood enters the right side of the heart, resulting in decreased right atrial, ventricular, and pulmonary artery pressures. At the same time, lower pressure in the systemic circulation further reduces cardiac work load by decreasing the peripheral vascular resistance against which the left ventricle must eject its blood. Afterload - as measured by aortic systolic pressure declines, and cardiac output increases.

50. Which of the following conditions necessitates a higher energy output (milliamps) for a temporary pacemaker? A. Digitalis toxicity B. Hypokalemia C. Atrioventricular block D. Ventricular dysrhythmias

Correct answer -B Hypokalemia necessitates a higher energy output for a temporary pacemaker, presumably because of its depressant effects on the myocardium. Myocardial ischemia or necrosis may cause the same problem. Necrotic tissue does not carry impulses, whereas ischemic tissue's capacity to respond to electrical stimulation is impaired.

42. The following blood gas samplings for Mr. A were obtained during Swan-Ganz insertion: right atrial 0 2 saturation, 75%; right ventricular, 94%; and pulmonary artery, 94%. This is diagnostic of: A. Atrial septal defect B. Cardiac tamponade C. Ventricular septal defect D. Pulmonary embolus

Correct answer -C Oxygen saturations in the right of the heart reflect deoxygenated blood returning from the body and are usually about 75% saturated. An abnormally high reading, such as that in the example, reflects a left-to-right shunt, with oxygenated blood returning to the right side of the heart. Because this abnormal reading starts in the right ventricle, a ventricular septal defect would be suspected.

179. Which of the following patients is most likely to be excluded from t-P A therapy? A. The patient was given streptokinase after a previous myocardial infarction with successful reperfusion B. The patient is 77 years old and on coumadin therapy at home c. The patient has had a recent streptococcal upper respiratory infection D. The patient has had chest pain for no more than 5 hours

Correct answer- B A 77-year-old patient on Coumadin therapy is not usually a candidate for thrombolytic therapy because of increased risk of bleeding from previous anticoagulant use - specifically intracranial bleeding in patients over age 70. Previous use of streptokinase and recent exposure to streptococcus bacteria are contraindications for strep tokinase (not t-PA) therapy, because of risk of allergic reaction Currently, thrombolytic therapy may be administered from up to 6 -12 hours after the onset of chest pain to preserve myocardial tissue.

69. Which patient is least at risk for developing cardiogenic shock? A. A 24-year-old with cardiac tamponade from a stab wound to the heart B. A 72-year-old with a history of heart disease who has developed a dysrhythmia C. A 55-year-old with acute pericarditis who has developed a dysrhythmia D. A 44-ycar-old with an anterolateral wall myocardial infarcation

Correct answer C The postoperative cardiac patient with acute pericarditis is least likely to develop cardiogenic shock. Cardiogenic shock is a vicious cycle that involves loss of myocardial contractility; resultant hemodynamic and metabolic reflex mechanisms cause further ischemia and the deterioration of contractility. Acute pericarditis, an inflammation of the pericardium, has a self-limiting, short-term clinical course when related to an acute myocardial infarction or cardiac surgery. The cause is not irritation but an autoimmune response lasting 2 to 6 weeks. Pericardial effusion usually does not develop. Pain, the most characteristic symptom of acute pericardiitis, usually can be alleviated by leaning forward. A pericardial friction rub is the most important physical finding. Most commonly, cardiogenic shock occurs as a complication of a myocardial infarction, especially when at least40% of the left ventricle is necrotic and ischemic. When the heart's ability to pump blood declines, the risk of cardiogenic shock increases. This occurs in conditions that produce deficient cardiac filling, such as cardiac tamponade and dysrhythmias in a compromised patient.

153. The most potent vasoconstrictor known is: A. Angiotensin II B. Bradykinin C. Renin D. Dopamine

Correct answer- A Angiotensin II is the most potent vasoconstrictor known. It produces arterial constriction and raises systolic and diastolic pressures. Renin is a protease secreted by the kidney; this secretion causes the kidney to release angiotensin I. Angiotensin I converts to angiotensin II by the alveolar cells of the lungs. Bradykinin is a polypeptide that induces vasodilation. Dopamine is the immediate precursor of norepinephrine. Classified as a vasopressor, dopamine in low doses may exert vasodilator effects on renal and mesenteric vessels.

180. Which laboratory value change may occur after t-PA therapy? A. Elevated fibrin split products B. Decreased fibrinogen C. Increased partial thromboplastin time D. Increased prothrombin time

Correct answer- A Because t-PA activates fibrin-bound plasminogen, the only Laboratory value change should be elevated fibrin split products. Increased prothrombin and partial thromboplastin times and decreased fibrinogen may occur with streptokinase therapy.

171. They physician orders an intravenous bolus of furosemide (Lasix) for Mr. L. The nurse feels that furosemide would be unsafe in this situation and that dopamine (Intropin) is indicated. If, after questioning the order, the nurse is told to give furosemide, she should: A. Refuse to give the furosemide B. Give the dopamine instead C. Give the furosemide D. Ask another physician

Correct answer- A If the nurse feels a medication to be unsafe, she should refuse to administer the medication, informing the physician and the nursing supervisor of this. The physician may administer the medication himself. If the setting of a right ventricular myocardial infarction, diuretics may worsen cardiac output and are containdicated. Inotm pic agents, such as dopamine (Intropin) and dobutamine (Dobutrex) may aid in improving cardiac output and are, in fact, indicated in the setting of a right ventricular myocardial infarction. Of course the nurse should carefully document all interactions with the physician.

81. Positive inotropic agents are used to: A. Improve tissue perfusion B. Decrease water loss through the kidney C. Increase heart rate D. Vasodilate vessels

Correct answer- A Inotropic agents increase the force of myocardial contraction, thereby improving tissue perfusion. A chronotropic agent increases heart rate.

182. An absolute contraindication to heart transplantation found during preoperative evaluation would be: A. Irreversible pulmonary hypertension B. Chronic obstructive pulmonary disease C. Recurrent ventricular dysrhythmias D. Diabetes mellitus

Correct answer- A Irreversible pulmonary hypertension is an absolute contraindication of heart transplantation since failure of the donor heart would occur immediately after transplantation. A heart-lungtransplantation would then be necessary if irreversible pulmonary hypertension is documented preoperatively Diabetes mellitus and chronic obstructive lung disease are relative contraindications, depending on the degree of impairment and disability. Recurrent ventricular dysrhythmias are actually an indication of heart transplantation.

121. The most common result of shock is: A. Metabolic acidosis B. Respiratory acidosis C. Metabolic alkalosis D. Hypovolemia

Correct answer- A Metabolic acidosis, or lactic acidosis, is the most common result of shock. The fall in cardiac output causes poor delivery of oxygen to the tissues. Because oxygen is necessary for cellular metabolism, the cells attempt to get energy by the anaerobic process of glycolysis, which produces tremendous amounts of lactic acid. Removal of carbon dioxide from the tissues also is impaired; tissue carbon dioxide levels then rise and react with water in the cells to produce carbonic acid.

109. Physiologic effects of cardiopulmonary bypass include all of the following except: A. Fever spikes B. Hemodilution c. Clotting abnormalities D. Third-space fluid shifting

Correct answer- A Physiologic effects of cardiopulmonary bypass include hemodilution, clotting abnormalities, and fluid shifts. Proteinuria may occur as a result of cardiopulmonary bypass. Decreased serum protein levels foster third-space fluid shifting into the interstitial spaces. Clotting abnormalities occur because the bypass machine is traumatic to the blood cells, especially the platelets. During the operation, platelet numbers drop to 40% to 50% of the preoperative level.

99. The nurse may observe pulsus paradoxus in patients with all of the following except: A. Patent ductus arteriosus B. Severe pulmonary emphysema C. Cardiac tamponade D. Hypovolemic shock

Correct answer- A Pulsus paradoxus is not seen in patent ductus arteriosus but may be seen in severe pulmonary emphysema, cardiac tamponade, pericardial effusions, and hypovolemic shock.

152. Which drug may prolong the QT interval? A. Isoproterenol (Isuprel) B. Atropine C. Disopyramide (Norpace) D. Epinephrine

Correct answer- C Disopyramide (Norpace) is a Type I antiarrhythmic similar to procainamid and quinidine. Norpace, as with other Type I antiarrhythmics, may prolong the QT interval. Isuprel, atropine, and epinephrine may shorten the QT interval by increasing the heart rate.

162. During pulmonary artery catheter placement, the correct lead to monitor Mr. V. is: A. Lead II B. Lead III C. Lead V1 D. Lead V6

Correct answer- C During pulmonary artery catheter placement, the patient should be monitored in lead V to observe for the complication of right bundle branch block.

151. The drug of choice for idiopathic hypertrophic subaortic stenosis (IHSS) is: A. Digoxin B. Inderal c. Lidocaine D. Dobutamine

Correct answer- B A heart with idiopathic hypertrophic sub aortic stenosis (IHSS) has an enlarged left ventricular septal wall, a reduced left ventricular outflow cavity, and a thickened and displaced mitral valve. Symptoms include breathlessness, weakness, and fatigue because of reduced compliance of the left ventricle. The patient may have frequent ischemic anginal attacks. The pharmacologic keystone of IHSS management is the beta-adrenergic blocking agent propranolol (Inderal). By specifically blocking the sympathetic receptors that stimulate heart rate, cardiac contractile force, and blood pressure, this drug automatically relieves ventricular obstruction. That is, by slowing the heart, it allows more time for diastolic filling and improves the ventricular volume vital to patients with obstructive cardiomyopathy.

101. A nursing responsibility during pericardiocentesis for cardiac tamponade is to: A. Prepare an infusion of isoproterenol (Isuprel) B. Monitor the patient's ECG tracings in a V lead C. Place the patient flat, if possible D. Encourage the patient to perform Valsalva's maneuver, if possible

Correct answer- B A precordial (V lead) ECG lead should be attached to the hub of the intracardiac needle. During pericardiocentesis, the ECG should be recording in the V lead. If the needle contacts the myocardium, an ST elevation will be recorded. During the procedure, a high Fowler's position will provide maximum comfort for the awake patient. Isoproterenol and Valsalva's maneuver are not indicated in the treatment of pericardia! tamponade.

148. An early P wave not followed by a QRS complex is known as: A. Sinus arrest B. Blocked premature atrial contraction C. Ventricular escape D. Second-degree A V block (Mobitz Type II)

Correct answer- B An early P wave not followed by a QRS complex is called a blocked premature atrial contraction. This usually occurs because the P wave is so early that the ventricle is still refractory from the previous impulse and cannot accept the new stimulus.

154. Which condition would stimulate renin secretion? A. Increased blood supply to the renal tubules B. Decreased blood pressure C. Decreased sympathetic output D. Increased sodium concentration

Correct answer- B Decreased blood pressure stimulates renin secretion. Other causes of renin secretion are increased sympathetic output and decreased serum sodium concentration. However, increases in blood supply to the renal tubules and in sodium concentration would inhibit renin secretion.

78. Dopamine is added to Mrs. C.'s therapeutic regimen. The therapeutic outcome of inotropic agents used to manage the patient in cardiogenic shock is: A. Vasodilation B. Improved tissue perfusion C. Increased heart rate D. Decreased force of myocardial contractility

Correct answer- B Inotropic agents improve tissue perfusion in the patient with cardiagenic shock. They increase the force of myocardial contraction: cardiac output increases because of enhanced contractility. Dopamine (Inotropin) is a sympathomimetic drug that has inotropic properties. Its vasopressor effects increase coronary blood flow, thereby improving tissue perfusion. The chronotropic effects of dopamine increase heart rate. This potential adverse effect limits its use because tachycardia increases myocardial oxygen consumption.

79. Which of the following medications would be most effective if prescribed for Mrs. C. to decrease preload and afterload? A. Dopamine B. Nitroprusside C. Dobutamine D. Digoxin

Correct answer- B Nitroprusside has a balanced vasodilating effect on both arteries and veins, so it effectively decreases afterload and preload. Dopamine, dobutamine, and digoxin are primarily positive inotropic agents; that is, their main effect is to increase the heart's contractility.

110. Because of rapid metabolic and temperature changes during the postoperative period, the nurse should check Mr.S. for: A. Dysrhythmias B. Heparin rebound C. Hyperkalemia D. Hyperthermia

Correct answer- B Postoperatively, the nurse should be alert for heparin rebound . The blood must be fully heparinized during extracorporeal circulation. Hypothermia, which develops during surgery, alters heparin metabolism. When the patient has been weaned off cardiopulmonary bypass, the heparin is reversed by the slow intravenous administration or protamine sulfate. Heparin reversal can cause patient complications, including heparin rebound. It is thought that heparin can return to the circulation postoperatively because the heparin and protamine are metabolized at different rates.

183. Signs of acute rejection Mrs. R. should be assessed for post-operatively include: A. Ischemia documented on ECG B. Lung crackles, gallop heart rhythm C. Hypertension, bradycardia D. Swelling at incision site

Correct answer- B Signs of congestive heart failure may indicate the development of acute rejection postoperatively. Assess for tachycardia, lung crackles, gallop heart rhythm, and hypotension. A fever may also develop.

169. The physician places a pulmonary artery catheter to guide therapy. Anticipated readings would be: A. RA pressure = 4 mm Hg; PCWP = 4 mm Hg B. RA pressure = 20 mm Hg; PCWP = 4 mm Hg C. RA pressure = 4 mm Hg; PCWP = 20 mm Hg D. RA pressure 20 mm Hg; PCWP = 20 mm Hg

Correct answer- B Since isolated right ventricle failure may occur, right atrial pressures are elevated. Since forward flow to the left side is decreased, pulmonary cpillary wedge pressures, as well as cardiac output, will be decreased.

76. While Mrs. C. is in cardiogenic shock, another beneficial effect of vasodilator therapy is: A. Increased systemic vascular resistance B. Decreased preload and afterload C. Increased pulmonary artery pressure D. Increased pulmonary vascular resistance

Correct answer- B The beneficial effect of vasodilator therapy in managing cardiogenic shock is decreased preload and afterload. Vasodilator therapy reduces venous tone and increases venous capacitance. This decreases blood return to the heart, thereby lowering the L VEDP. Vasodilator therapy also reduces arterial vascular tone, which results in decreased impedance to left ventricular ejection (afterload); cardiac work load decreases, and cardiac output increases.

41. Afterload is indicated best by the measurement of: A. Pulmonary capillary wedge pressure B. Stroke volume.................................. C. Systemic vascular resistance.......... D. Ejection fraction...............................

Correct answer- C Measurement of systemic vascular resistance best indicates afterload. The higher the pressure in the systemic circulation, the greater the impedance of ejection (afterload). The pulmonary capillary wedge pressure indirectly reflects left ventricular end-diastolic pressure, referring to preload. Stroke volume is the amount of blood ejected by the ventricle with each cardiac contraction, referring to contractility. Ejection fraction is the percentage of the left ventricular end-diastolic volume ejected during each cardiac cycle.

161. Although initially stable upon admission, Mr. V.'s blood pressure and urine output have decreased over the last two hours. The physician decides to place a pulmonary artery catheter to aid in hemodynamic assessment, obtains consent and leaves the room to set up. Mr. V. begins to ask questions, which leads the nurse to conclude that he does not understand the information just given by the physician. The nurse should: A. Answer Mr. V.'s questions B. Notify the supervisor C. Notify the physician D. Do nothing because the consent is already signed

Correct answer- C If, after the physician has left, a patient does not understand the in formation just given by the physician during an informed consent, the nurse should notify the physician, who should return and answer the patient's questions. If the physician does not return, the nurse's supervisor should be called and the procedure delayed until further explanations are rendered. The nurse should not answer the patient's questions because she is then assuming the physician's role and expertise. If the patient has questions that the nurse is not a liberty to answer, an informed consent was probably not obtained.

117. Which of the following nursing actions is inappropriate during the initial stabilization of Mr. M.? A. Administering oxygen therapy B. Applying medical antishock trousers (MAST) C. Placing the patient in the Trendelenburg position D. Infusing lactated Ringer's solution via a large-bore I.V. line

Correct answer- C Initial management of a patient in shock involves provisions for airway and ventilation; maintenance of blood volume, pressure, and circulation; and maintenance of cellular oxygen consumption. Specific measures include oxygen therapy and administration of intravenous fluid, preferably lactated Ringer's solution via a large-bore I.V. line. Medical antishock trousers (MAST) can also be used in patient with leg or abdominal injuries who have signs and symptoms of hypovolemic shock. This will autotransfuse about 1,000 mI of blood, reduce blood loss by application of direct pressure , and immobilize leg injures. The patient in shock should be flat, with only the legs elevated. The Trendelenburg position, once favored in managing shock, has been abandoned because it allows the diaphragm to migrate upward, thus compromising ventilation. Also, this position may cause a reflex inhibition of the pressoreceptor activity, thereby decreasing sympathetic stimulation and further compromising arterial blood pressure.

108. Mr.S. undergoes CABG. Postoperatively, the nurse would assess him for complications. Which would be least likely to occur? A. Graft occlusion B. Perioperative infarction C. Numbness of donor leg D. Progression of atherosclerotic process

Correct answer- C Numbness of the donor leg would be least likely to occur postoperatively in the patient who has undergone CABG. Complications would include graft occlusion, perioperative infarction (which occurs in 2% to 4% of patients postoperatively), and progression of the atherosclerotic process, because saphenous vein grafts are prone to atheromatous changes.

181. Which of the following indicates successful reperfusion after t-PA therapy? A. Negative creatine phosphoskinase (CPK) isoenzymes B. Q wave formation C. Atrial fibrillation D. Accelerated idioventricular rhythm

Correct answer- D Noninvasive markers of successful reperfusion include relief of chest pain, normaliziation of ST segments, reperfusion dysrhythmias, and rapid rise of creatine phosphokinase (CPK) isoenzymes. Reperfusion dysrhythmias include accelerated idioventricular rhythm, sinus bradycardia, premature ventricular ectopy, and ventricular tachycardia. A rapid rise of CPK isoenzymes occurs because the reperfused artery washes out the CPK enzymes, which are then released into the systemic circulation, after the onset of chest pain in order to preserve myocardium.

187. Which statement by Mrs. R. demonstrates correct under standing regarding her heart rate/rhythm post-transplant? A. "There should be no change in my heart rate or rhythm because of the transplant" B. "I have a pacemaker now to help my heart beat effectively" C. "My heart beats more rapidly because of irritability from scar tissue" D. "My heart rate rises more slowly now when I begin activity or exercise"

Correct answer- D Because there is no direct connection between Mrs. R. 's native sinus pacemaker and her heart, her heart rate does not rise quickly in response to increased metabolic demands, such as exercise, activity or fever. Instead, a slow rise in heart rate will be seen due to an increase in circulating catecholeamines. Mrs. R. should be instructed to begin activity or exercise slowly to compensate for this change in physiology.

Case Study: Ms. N, 68, is admitted to the ICU with chest pain. She has a history of angina and ventricular dysrhythmia. Two weeks ago she was started on quinidine gluconate. Questions I 7 4 - 177 refer to the above situation.

Correct answer- D Deep T wave inversion in leads V2-V3 is consistent with Wellen's syndrome, which indicates a critical stenosis of the left anterior descending coronary artery. Associated signs inlcude little or no enzyme elevation, no Q wave formation, and little or no ST elevation. Wellen's syndrome is a warning sign for an impending acute anterior wall myocardial infarction.

77. The effectiveness of vasodilator therapy may be most immediately indicated by: A. Pa02 B. Vital signs C. PaC02 D. Urine output

Correct answer- D Effective vasodilator therapy improves cardiac output. Increased urine output reflects a generalized improvement in blood flow to the tissues, including the kidneys, and a reversal of the shock syndrome.

71. Clinical manifestations of Mr. C.'s cardiogenic shock would include all of the following except: A. Distended neck veins B. Pulmonary congestion C. s3 heart sound D. Low central venous pressure

Correct answer- D The clinical manifestations of cardiogenic shock result from the heart's impaired contractility, which decreases cardiac output and elevates left ventricular end-diastolic pressure (L VEDP). The reduced cardiac output impairs perfusion to the vital organs and peripheral tissues. Thus, the patient exhibits changes in sensorium, such as restlessness, anxiety, or confusion. The skin is cold and clammy from peripheral vasoconstriction. As the cardiac output declines, the pulse pressure narrows; peripheral pulses are rapid and thready. Systemic arterial pressure drops, and hypotension ensues. Urine output decreases as kidney perfusion declines. A rise in the L VEDP increases left atrial and pulmonary pressures, resulting in pulmonary congestion. Tachypnea, dyspnea, and pulmonary edema develop; an S3 can be heard as the left ventricle fails and becomes overdistended during diastole. Increased pressures on the left side of the heart progress back through the pulmonary venous system into the right side of the heart. This increase in pulmonary pressure decreases the right ventricular performance and may result in right ventricular failure, which in turn produces an elevated CVP and jugular vein distention with hepatomegaly.

131. Which finding in Mr. B. would not indicate reperfusion during t-PA infusion? A. Drop in arterial blood pressure B. Resolution of ST segment elevation C. Ventricular tachycardia D. Dramatic reduction in chest pain

Correct answer-A Findings that indicate reperfusion during t-PA infusion include dramatic reduction in chest pain, resolution ofST segment elevation, and reperfusion dysrhythmias. These dysrhythmias include sinus bradycardia, accelerated idioventricular rhythm, ventricular tachycardia, heart block, and ventricular fibrillation.

132. Which of the following is not a complication of an acute myocardial infarction (MI)? A. Myocarditis B. Dressler's syndrome C. Ventricular aneurysm D. Papillary muscle rupture

Correct answer-A Myocarditis is not a complication of an acute myocardial infarction. Myocarditis is an inflammatory disease ofthe myocardium and may be viral, protozoal (Chagas disease), or immunologic. Dressler's syndrome, a postinfarction pericarditis occurring in 2% to 5% of patients 1 to 4 weeks post-MI, is characterized by fever and pericardia! effusion. Ventricular aneurysm and papillary muscle rupture are consequences of extensive left ventricular damage.

128. The nurse administering t-P A must monitor Mr. B. for all of the following except: A. Peripheral thrombosis B. Myocardial reperfusion C. Bleeding complications D. Coronary reocclusion

Correct answer-A The nurse administering tissue plasminogenactivator(t-PA) therapy must monitor the patient for myocardial reperfusion because t-PA opens closed coronary arteries about 70% of the time. Bleeding complications may occur because t-PA causes fibrin clot dissolution and coronary reocclusion. Peripheral thrombosis is not associated with t-PA therapy.

133. Suddenly, Mr.F. develops ventricular tachycardia. He is pulseless. Your immediate response would be to: A. Defibrillate with 200 joules B. Defibrillate with 300 joules C. Administer lidocaine 100- mg bolus D. Perform a precordial thump

Correct answer-A Ventricular tachycardia without a pulse should be treated similarly to ventricular fibrillation- by immediate unsynchronized defibrillation at 200 joules for the first attempt, 300 joules for the second, and 360 joules for the third. A precordial thump should be given for pulseless ventricular tachycardia only when a defibrillator is unavailable: although this maneuver may occasionally convert ventricular tachycardia to sinus rhythm, it is equally likely to convert the tachycardia to ventricular fibrillation.

129. Before initiating t-PA therapy, the nurse must establish a mini mum of how many intravenous infusions? A. One B. Two C. Three D. Four

Correct answer-B Before initiating t-PA therapy, the nurse must establish a minimum of two intravenous infusions: the t-PA infusion, and a continuous heparin infusion initiated during the first hour of t-PA therapy.

88. A patient with typical anginal pain such as Mr. A.'s may describe it as: A. A vague ache over the left side of the chest B. A tight, oppressive substernal pain radiating to the left arm C. A severe, sharp pain around the sternum radiating up both sides of the neck D. An intense, shooting pain from the left side of the chest radiating to the left arm

Correct answer-B Clinical presentation in the patient experiencing angina pectoris is precordial, middle, or lower sternal pain that may radiate to the shoulders, arms, neck, and jaw. The pain is usually described as burning, squeezing or heavy, not sharp or shooting. Characteristically, the pain begins on exertion, usually lasts 1 to 4 minutes, and subsides when precipitating factors are removed.

123. Which of the following is a consequence of multiple transfusions? A. Brodzinski's sign B. Chvostek' s sign C. Cullen's sign D. Babinski's sign

Correct answer-B Hypocalcemia is a potential problem during massive transfusions because citrate preservatives may bind ionized calcium. In fact, alkalosis may also develop after multiple transfusions, as sodium citrate is converted to sodium bicarbonate. Patients who have these electrolyte and acid-base imbalances may demonstrate Chvostek's sign, a facial twitching that occurs when the side of the face is tapped. Brudzinski's sign indicates meningeal irritation; both the upper legs at the hips and the lower legs at the knees are flexed in response to the path of flexion of the head and the neck on the chest. Babinski's sign, which is the dorsiflexion of the big toe upon scratching the bottom of the foot, indicates upper motor neuron dysfunction. Cullen's sign, ecchymosis around the umbilicus, usually is seen in hemorrhagic pancreatitis.

125. A thrombolytic agent used in the first few hours of an MI is: A. Heparin B. Streptokinase (Streptase) C. Thrombokinase D. Warfarin (Coumadin)

Correct answer-B Streptokinase is a nonenzymatic protein thrombolytic agent excreted by group C beta-hemolytic streptococci. Thrombolytic therapy is effective in treating massive pulmonary emboli, extensive deep vein thrombosis, and lysis of a thrombus-occluded arteriovenous cannula. Substantial evidence supports the use of streptokinase in treating acute MI in its early stages. Heparin and warfarin are anticoagul ant agents. Thrombokinase does not exist.

130. The total dose of t-PA, as approved by the Food and Drug Administration (FDA), is: A. 50 mg B. 100 mg C. 200 mg D. 500 mg

Correct answer-B The total dose of t-P A, as approved by the Food and Drug Administration (FDA), is 100 mg. This dose is infused over 3 hours as follows: 60 mg over the first hour, with6to 10mg given as an l.V. bolus over 1 to 2 minutes; 20 mg over the second hour; and 20 mg over the third hour.

127. Mr. B.'s thrombolytic therapy should be discontinued if which of the following occurs? A. Low-grade fever B. Vomiting C. Dyspnea D. Relief of chest pain

Correct answer-C Thrombolytic therapy should be discontinued immediately if the patient develops dyspnea, which may indicate a severe allergic reaction. Adverse reactions to streptokinase therapy include hemorrhagic complications with bleeding at I. V. sites, incision sites, and wounds. These can be treated with pressure and discontinuation oftherapy. In severe hemorrhage, fresh frozen plasma can be administered and aminocaproic acid (Ami car) may be needed. Allergic reactions include urticaria, pruritus, headache, musculoskeletal pain, bronchospasm, edema, hypertension. and fever. Febrile reactions and mild allergic symptoms can be treated with acetaminophen, antihistamines, and steroids. Severe allergic reactions warrant discontinuation of the infusion. Rare adverse effects include delirium and tachycardia.

87. IABP therapy is contraindicated for which of the following? A. Papillary muscle rupture B. Incompetent aortic valve C. Left ventricular failure after bypass D. Unstable angina refractory to the medical regimen

IAPB therapy is contraindicated in a patient with an incompetent aortic valve because balloon inflation would increase aortic regurgitation. Papillary muscle rupture causes mitral valve incompetence. Decreasing afterload through the use of IABP may save the patient's life until surgical repair of the mitral valve occurs. In left ventricular failure after bypass, IABP therapy provides circulatory assistance until recovery occurs. This method has significantly reduced postoperative mortality. IAPB therapy is also indicated for unstable angina refractory to the medical regimen. Increased coronary artery perfusion stops the angina promptly, increasing the ejection fraction and thereby stabilizing the patient.

89. All of the following are true regarding angina pectoris except: A. Pain may radiate to shoulders, arms, or jaw B. Relief from nitroglycerin may take as long as 1 hour C. Pain gradually subsides with rest D. ST depressions may be seen during an attack

Nitroglycerin usually relieves the pain of angina pectoris within 45 to 90 seconds. If not, angina may have progressed to the unstable state or an infarction may be occurring.

106. Which of the following findings would not be a contraindication for CABG when selecting a candidate? A. Poor left ventricular function B. Recent myocardial infarction C. Distal coronary artery disease D. History of an old MI

Poor left ventricular function, a recent myocardial infarction, and distal coronary artery disease are contraindications for CABG. A history of an old MI is not a contraindication for this surgery.

83. Which of the following is true concerning the IABP? A. Increases left ventricular pressure B. Increases PCWP C. Increased coronary artery perfusion D. Increases afterload

The intra-aortic balloon pump (IABP) increases coronary artery perfusion. When the balloon is inflate, it occludes about 80% of the thoracic aorta, thereby displacing the blood up toward the coronary arteries. Because the heart muscle derives its blood supply during diastole, balloon inflation provides more blood to the myocardium during diastole, thus increasing coronary artery perfusion.


Kaugnay na mga set ng pag-aaral

macro: required reserves, excess reserves, and bank behavior

View Set

Help Desk Customer Service Quiz #6(slides part two)

View Set

Unit 1, Concept 3 --> Energy Flow

View Set